Anda di halaman 1dari 64

UNIVERSITY PAPER SOLUTION YEAR- 2009

Q-1: Explain briefly the various stages of management control process citing salient features of each. Ans: Management Control is the process by which managers influence other members of the organization to implement the organizations strategies. Following are the various stages of Management Control Process: 1) Programming 2) Budgeting 3) Execution 4) Evaluation Control process in the of non-operating activities such as project consist of the above phases except that two phases, programming and budgeting are combined into a single activity. A project generally has a single objective and on-going operating activities have multiple objective. A project comes to an end when the objective is accomplished. The Stages are discussed below: 1) Programming: Programming is defined as making programs by top/senior management in terms of organization, goals and strategies and deciding the fund and resources needed to accomplish the programs. Programs can be made about development of new products, research and development activities, merger takeover, and other activities that are not related much with existing product lines. In service organizations, such as a hotel, chain management may draw programs for each hotel on each region where the hotels are to be set up. Programming is a long rang plan, covering period of approximately five future years. The reason is that if programming is made for shorter periods, the result and benefits of programming cannot be realized within this period. Some organization like public utilities, prepare long rang plans for even a periods of twenty years. Because of relatively long time plan, only rough estimates are possible for revenues, expenses and capital expenditure.

Following is the Criticality of Programming: a) The top management is convinced that programming is very important. Otherwise programming is likely to become a staff exercise that has little impact an actual decision making. b) The Organisation is relatively a large and complex. In small, simple organization, an informal understanding of an organizations future direction is adequate for making decision about resources allocation, which is the principle purpose of repairing programme. c) Considerable uncertainty about the future exists in the organization, but organization has the flexibility to adjust to change the circumstances. In relatively stable organization, a program may be unnecessary; the future is sufficiently like the past so that the program would be only an exercise in extrapolation. If the future is so uncertain that reasonably reliable estimates cannot be made, preparation of formal programme is a waste of time. 2) Budgeting: Budgeting is formal financial plan for each year. It is known as short range plan. It is a technique of expressing revenues, expenses, physical target like production and sales, profit, asset and liabilities usually for periods of one future year. Budget has functions of motivating manager, coordinating activities, communicating to persons within an organization, providing standards for judging actual performance and acting as a control tool. 3) Executing:After the budget preparation, budgeting is used as tool for coordinating the action of individuals and departments within the organization. In fact within the execution phase, task control is done to ensure the action and performance match with the end desired result. While managers goals is to achieve budgeted targets, however compliance to budget is not necessary if the plans given in the budget are found as not the best way of achieving the objectives. Adherence to budget is not necessarily good, and departure from it is not necessarily bad. 4) Evaluation: The management control process ends with evaluation phase in which performance of manager is evaluated. Since it is an after event exercise, the evaluation does affect what has happened. However evaluation phase acts like a powerful stimulus, as employees know that their performance will be subsequently evaluated. Also on the basis of performance evaluation, the future budget and plans are revised.
3

The management control process is behavioural, manifesting itself in interaction among managers and between managers and their sub-ordinates. Because managers differ from another in technical ability, leadership style, interpersonal skill, experience, approach to decision making, affinity for member, and in many other ways, the details of the management control process vary from company to company and among the responsibility centres within company. The difference relates mainly to the way the control system is used. Programming, budgeting, executing and evaluation are not needed in small, relatively stable organizations, and it is not worthwhile in organizations that cannot make reliable estimates about the future or in organizations whose top management does prepare to manage in this fashion. Q-2: What is Responsibility Centre? List and explain different types of responsibility centres with sketches. Ans: A responsibility centre may be defined as an area of responsibility which is controlled by an individual. . A responsibility centre is an activity such as a department over which a manager exercises responsibility. Responsibility areas may be departments (drilling or maintenance department), product lines (chemicals or fertilizers), territories (North or South) or any other type of identifiable unit or combination of units. The specific types of responsibility areas depend on the nature of the firm and its activities. It is relatively easy to identify activities with specific managers. A plant manager is in charge of a plant and is usually responsible for producing budgeted quantities of specific products within budgeted cost limit. A sales manager is responsible for getting orders from customers, and so on. A responsibility centre exists to accomplish one or more purposes, termed as its objectives. The objectives of the companys various responsibility centres are to help implement these strategies. Types of Responsibility Centres: Responsibility centres can be classified by the scope of responsibility assigned and decision-making authority given to individual managers. The following are four common types of responsibility centres: 1) Cost Centre: A cost or expense centre is a segment of an organization in which the mangers are held responsible for the cost incurred in that segment but not for revenues. Responsibility in a cost centre is restricted to cost. For planning purposes, the budget
4

estimates are cost estimates; for control purposes, performance evaluation is guided by a cost variance equal to the difference between the actual and budgeted costs for a given period. Cost centre managers have control over some or all of costs in their segment of business, but not over revenues. Cost centres are widely used forms of responsibility centres. In manufacturing organizations, the productions and service departments are classified as cost centre. Also, a marketing department, a sales region or a sales representative can be defined as a cost centre. Cost centre may vary in size from a small department with a few employees to an entire manufacturing plant. In addition cost centres may exist within other cost centres. For example, a manager of a manufacturing plant , with the department with a few employees to an entire manufacturing plant organized as a cost centre may treat individual departments within the plant as separate cost centres, with the department managers reporting directly to plant manager. Cost centre managers are responsible for the costs that are controllable by them and their subordinates. However, which costs should be charged to cost centres, is an important in evaluating cost centre managers. 2) Revenue Centre: A revenue centre is a segment of the organization which is primarily responsible for generating-sales revenue. A revenue centre manager does not possess control over cost, investment in assets, but usually has control over some of the expenses of the marketing department. The performance of a revenue centre is evaluated by comparing the actual revenue with budgeted revenue and actual marketing expenses. The Marketing manager of a product line or an individual sales representative is examples of revenue centres. For e.g. In 1999 two companies, Service and Impact Hotel Group, merged to create Lodgian, Inc., one of the largest owners and operators of hotel in the United States. Lodgian reorganized itself into six regions, each with a Regional Vice-president, a regional operational manager, and a regional Director of sales and marketing. The sales and marketing functions were constituted as revenue centres, with the goal to significantly improve market share. In the highly competitive call centre industry environment of 2004, some companies successfully differentiated themselves by converting their services centres into revenue centres. The revenue streams were generated through after service sales. The call centre agents would address the calling customers needs and requests, provide the necessary
5

service, and then offer some type of new product or service that would meet the customer needs. 3) Profit Centre: A profit centre is a segment of an organization whose manager is responsible for both revenues and costs. Managers of profit centres have control over both costs and revenues. . In a profit centre, the manager has the responsibility and the authority to make decisions that affect both costs and revenues for the department or division. The main purpose of a profit centre is to earn profit. Profit centre managers aim at both the production and marketing of a product. The performance of the profit is evaluated in terms of whether the centre has achieved its budgeted profit. A division of the company which produces and markets the products may be called a profit centre. Such a divisional manager determines the selling price, marketing programmes and production policies. Profit centres make managers more concerned with finding ways to increase the centres revenue by increasing production or improving distribution methods. The manager of a profit centre does not make decisions concerning the plant assets available to the centre. For e.g. the manager of the sporting goods department does not make the decisions to expand the available floor space for the department. Mostly profit centres are created in an organization in which they sell products or services outside the company. In some cases, profit centres may be selling products or services within the company. For example, repairs and maintenance department in a company can be treated as a profit centre if it is allowed to bill other production department for the services provided to them. Similarly, the data processing department may bill each of companys administrative and operating departments for providing computer related services. 4) Investment Centre: An investment centre is responsible for both profits and investments. The investment centre manager has control over revenues, expenses and the amount invested in the centre assets. He also formulates the credit policy which has a direct influence on debt collection, and the inventory policy which determines the investment in inventory. The manager of an investment centre has more authority and responsibility than the manager of either a cost centre or a profit centre. Besides controlling costs and revenues, he has investment responsibility too. Investment on asset responsibility means the authority to buy, sell and use divisional assets. For e.g: division of a large multinational Company. The division is assessed in terms of its contribution to overall profits.
6

Q-3: Every SBU is a profit center but every profit center is not a SBU. What are the conditions that should be fulfilled for an organization unit to be converted into a Profit Center? What are the different ways to measure the performance of Profit Centers? Discuss their relative merit and demerits. Ans: In the competitive market environment of todays business cannot survive unless there is total accountability and associated responsibility and authority. Distribution sector also needs to be treated as a business entity if financial viability is to be achieved. The heads of the business units should be empowered to act and be held accountable for their actions & performance. Such a concept would be achievable if each circle is declared as a profit center with its own accounting system. The performance parameters as well as benchmarks can be set for improvement. This would also bring in the sense of ownership and competition, which are essential ingredients for success of a business. The MOA stresses upon the need for declaration of a circle as a profit center and establishing base line parameters as well as bench marks for measuring improvements consequent upon the commercial, administrative and technical interventions. The operating expenses of the circle, which contribute towards the delivery cost of energy to the customer, can also be monitored more closely as a profit center concept and measures may be initiated for reduction of the same. Most business units are created as profit centers since managers in charge of such units typically control product development, manufacturing & marketing resources. These managers are in a position to influence revenues and costs and as such can be held accountable for the bottom line. However, a business unit managers authority may be constrained in various ways, which ought to be reflected in a profit centers design and operation. Functional units Multi-business companies are typically divided into business units, each of which is treated as independent profit-generating units. The subunits within these business units however, may be functionally organized. It is sometimes desirable to constitute one or more of the functional units e.g., marketing, manufacturing & service operations as profit centers. There is no guiding principle declaring that certain types of units are inherently profit centers and others are not. Managements decision as to whether a given unit should be a profit center is based on the amount of influence the units managers exercises over the activities that affect the bottom line.
7

Conditions for an organization unit to be converted into a profit center: Functional organization is one which each principal manufacturing or marketing function is performed by a separate organization unit. When such an organization is converted to one in which each major unit is responsible for the manufacture and marketing, the process is termed divisionalization. As a rule, companies create business units because they have decided to delegate more authority to operating managers. Although the degree of delegation may differ from company to company, complete authority for generating profits is never delegated to a single segment of the business. Many management decisions involve proposals to increase expenses with the expectation of an even greater increase in sales revenue. Such decisions are said to involve expense/revenue trade-offs .Additional advertising expense is an example. Before it is safe to delegate such a trade-off decision to a lower-level manager, two conditions should exist: 1) The manager should have access to the relevant information needed for making such a decision. 2) There should be some way to measure the effectiveness of the trade-offs the managers has made. A major step in creating profit centers is to determine the lowest point in an organization where these two conditions prevail. All responsibility centers fit into a continuum ranging from those that clearly should be profit centers to those that clearly should not, management must decide whether the advantages of giving profit responsibility offset the disadvantages, which are discussed below .As with all management control system design choices; there is no clear line of demarcation. Different ways to measure performance of profit centers: The classification and establishment of responsibility center may help an organization to get better performance. However, if we must determine the performance of each responsibility center, we should take proper means to measure and evaluate it. Center performance measurement is the process of accumulating and reporting data related to center performance. The performance report includes financial data, operating statistics considered important to performance, and operations budget for evaluation basis.
8

Performance evaluation is the judgment process of supervisors about the quality of the performance of subordinates. The results of performance evaluation are qualitative judgments such as outstanding, good, adequate, or poor. The evaluation takes the forms of a memorandum that will provide part of the basis for salary increases, bonuses, and future promotions. Performance measures are the relatively objective numbers resulting from a performance measurement system, and performance evaluations are the subjective judgment of managers. If the evaluations are fair and reasonable, there should be some correspondence between the measures and the judgments. Because the responsibility and authority of each center are different, the measurement approaches of performance of each center are different. In the following section, we discuss performance measures in responsibility centers. A profit center may determine which products and how many products should be produced and sold. But it only controls costs and revenues related to the profit center other than those of the entire organization. Thus, the performance of profit center that is gained by means of its profit that is gained by means of controlling its operational activity, actually cannot be real profit of the organization, but only be contribution to profit of the organization or its higher level responsibility center. Often, profit centers are evaluated by means of contribution margin income statements, in terms of meeting revenues and costs objectives. This mainly takes form of controllable income to measure. Controllable income is the excess of contribution margin over fixed costs controlled by the profit center. Contribution margin is the excess of revenue of the profit center over all variable costs of those sales. We may measure the performance of the profit center or its manager by means of the controllable income variance that is the difference between the actual and planned number of the controllable income. However, in profit centers, we encounter the usual problems related to measuring profit for the organization as a whole: how are the organizations revenues and costs allocated to each profit center? If a profit center is totally separate from all other parts of the organization, its profits can be uniquely identified with it. However, most profit centers have costs (and perhaps revenues) in common with other units. The organization faces a cost allocation problem.
9

A related problem involves the transfer of goods between a profit center and other parts of the organization. Such goods must be priced so that the profit center manager has incentives to trade with other units when it is in the organization's best interests. The organization faces this transfer-pricing problem. It is not easy to determine how to measure performance in a profit center exist. No matter what process is chosen, its objectives should be straightforward: Measure employees' performance in ways that motivate them to work in the best interest of their employers and compare their performance to standards or budget plans. ADVANTAGES: A profit centre is that segment of activity of a business which is responsible for both revenues and expenses and discloses the profit of a particular segment of activity. It is created as a result of decentralization of operations to measure the performance of divisional executives. Each profit centre has a profit target and also enjoys authority to adopt such policies as are necessary to achieve its targets. 1) The chief merit of profit centre is that it makes its managers responsible for the profit performance achieving the budgeted amount of profit during a period. 2) Under profit centre concept, the whole organization is divided into a number of divisions; the performance of each division is measured in terms of both the income that is earned and the costs that are incurred. 3) Headquarters management, relieved of day to day decision making, can concentrate on broader issues. 4) Managers in each division have freedom in making decisions. They need not obtain approval from corporate headquarters for every expenditure. 5) The quality of decisions may improve because they are being made by managers who are closet to the point of decision. 6) The speed of operating decisions may be increased, since they do not have to be referred to corporate headquarters.

10

DISADVANTAGES: 1) Divisions may compete with each other and may take decisions to increase profits at the expenses of other divisions, thereby overemphasizing short term results. 2) It may adversely affect co-operation between the divisions and lead to lack of harmony in achieving organizational goals of the company. Thus it is hard to achieve the objective of goal congruence. 3) It may lead to reduction in the companys overall total profits. 4) The cost of activities which are common to all divisions may be greater for decentralized structure than for centralized structure. It may thus result in duplication of staff activities. 5) Top management loses control by delegating decision marking to divisional managers. These are risks of mistakes committed by the divisional managers which the top management may avoid. 6) Series of control reports prepared for several departments may not be effective form the point of view of top management. 7) It may underutilize corporate competence. 8) It leads to complication associated with transfer pricing problems. 9) It becomes difficult to identify and define precisely suitable profit centers. 10) It confuses divisions result with managers performance. Q-4 a) Transfer Pricing is not an Accounting Tool. Comment with illustrations. Ans:Transfer pricing refers to the amount used in accounting for any transfer of goods and services between responsibility centres. This is a narrow definition and limits the term transfer price to the value placed on a transfer of goods or services in transaction in which at least one of the two parties is involved in the profit centre. Such a price typically includes a profit element because an independent company normally would not transfer goods or services to another independent company at cost or less.

11

Therefore, the mechanism for allocating cost in an accounting system; such cost do not include a profit element. The term price as used here has the same meaning as it has when used in connection transaction between independent companies. OBJECTIVE OF TRANSFER PRICING: Profit centres are responsible for product development, manufacturing and marketing each share in the revenue generated when the product is finally sold. The transfer price should be designed so that it accomplishes the following objectives: 1) It should provide unit with the relevant information it needs to determine the optimum trade-off between company cost and revenues. 2) It should induce goal congruence decision i.e., the system should be designed so that decision that improve business unit profit will also improve company profits. 3) It should help to measure the economic performance of the individual business units. 4) The system should be simple to understand and easy to administer. Thus, from the objective, it is understandable that the Transfer price is mainly transferring of goods and services from one unit to another, where much important is not given to accounting basis but also to all other effects.

12

Q-4b) Market price is ideal transfer price even in Limited Markets. Comment. Ans:A market price-based transfer price will induce goal congruence if all the following conditions exist. Rarely, if ever, will all these conditions exists in practice. The list, therefore, does not set forth criteria that must be met to have a transfer price. Rather, it suggests a way of looking at a situation to see what changes should be made to improve the operation of the transfer price mechanism. 1) Competent people: Ideally, managers should be interested in the long-run as well as the short-run performances of their responsibility centres. Staff people involved in negotiation and arbitration of transfer price also must be competent. 2) Good Atmosphere: Managers must regard profitability, as measured in their income statements, as an important goal and a significant consideration in the judgment of their performance. They should perceive that the transfer prices are just. 3) A Market Price: The ideal transfer price is based on a well-established, normal market price for the identical product being transferred-that is, a market price reflecting the same conditions (quantity, delivery time, and quality) as the product to which the transfer price applies. The market price may be adjusted downward to reflect savings accruing to the selling unit from dealing inside the company. For e.g. there would be no bad debt expense, and advertising and selling costs would be smaller, when products are transferred from one business unit to another within the company. Although less than ideal, a market price for a similar, but not identical, product is better than no market price at all. 4) Freedom to source: Alternatives for sourcing should exist, and managers should be permitted to choose the alternative that is in their own best interests. The buying managers should be free to buy from the outside, and the selling manager should be free to sell outside. In these circumstances, the transfer price policy simply gives the manager of each profit center the right to deal with either insiders or outsiders at his/her discretion. The market thus establishes the transfer price. The decision as to whether to deal inside or outside is also made by the marketplace. If buyers cannot get a satisfactory price from the inside source, they are free to buy from the outside.

13

This method is optimum if the selling profit centre can sell all of its products to either insiders or outsiders and if the buying centre can obtain all of its requirements from either outsiders or insiders. The market price represents the opportunity costs to the seller of selling the product inside. This is so because the product was not sold outside. From a company point of view, therefore, the relevant cost of the product is the market price because that is the amount of cash that has been forgone by selling inside. The transfer price represents the opportunity cost to the company. 5) Full Information: Managers must know about the available alternatives and the relevant costs and revenues of each. 6) Negotiation: There must be a smoothly working mechanism for negotiating contracts between business units. If all of these conditions are present, a transfer price system based on market prices would induce goal congruent decisions, with no need for central administration. Q-6: Enumerate the differences among following types of Audits: a) Financial Audit (Statutory) b) Cost Audit c) Efficiency Audit d) Management Audit. Ans:a) Financial Audit (Statutory): A financial audit, or more accurately, an audit of financial statements, is the review of the financial statements of a company or any other legal entity (including governments), resulting in the publication of an independent opinion on whether or not those financial statements are relevant, accurate, complete, and fairly presented. Financial audits are typically performed by firms of practicing accountants due to the specialist financial reporting knowledge they require. The financial audit is one of many assurance or attestation functions provided by accounting and auditing firms, whereby the firm provides an independent opinion on published information. Following are the features of Financial Audit:
14

1. Simplified input of auditing tasks (audit sheets, recommendations and action plans) 2. Instant information access and sharing for everyone. 3. Unified auditing methods. 4. Automated report generation. 5. Less labour intensive and time-saving during report review meetings. b) Cost Audit:Cost Audit is the verification of cost accounts and check on adherence to the cost accounting plan. Cost Audit is mainly a preventive measure, a guide for a monetary policy and decision, in addition to being a barometer of performance. Cost Audit is useful because it provides information/ data: On Cost of Production For Price Fixation For Arriving at the Standard cost of the product Identifying the Inefficiencies in different Departments It helps in identifying the weaknesses in the system and also pinpoints the inefficient activities to different departments. It also aims to simplify the wastages and losses which can be avoided. c) Efficiency Audit: Efficiency Audit is concerned with the allocation of resources to different uses in the business and efficient utilisation of resources allocated to each use. The Cost Auditor helps the top management in financial planning, performance evaluation, and efficiency in operations and in establishing coordination between departments. Efficiency Audit aims at ensuring that: Every rupee invested in capital or in other field yields optimum return Investment in different functions and aspects of the business has been so balanced so that the return on investment is optimum. d) Management Audit: The Management Audit is total examination of an organisation or a part of it. It Includes:

15

Check on the effectiveness of managers of their compliance with the Company or Professional Standards The reliability of Management data. The quality of Performance of duties Recommendations for Improvement CIMA Defines Management Audit asAn objective and independent appraisal of the effectiveness of the corporate culture in the achievement of company objectives and policies. Following are the objectives of Management Audit: To ensure effective utilisation of resources. To identify deficiencies in policies and procedures. To suggest improvement in methods of operation. To analyse internal controls and suggest improvements if any To identify the need for restructuring and suggest ways and means for the same To anticipate managerial problems and evolve mechanism to handle them effectively. Q-8: What do you understand by Goal Congruence? What are the informal factors that influence goal congruence? Ans: Each individual has his personal goals. He joins an organization to achieve then goals. The personal goal may just be to get a job that assures safety and monetary rewards. The organization, through its top management, sets for itself pals that are desired to achieve. At times there is a conflict between individual pals and organizational goals. Such conflict is more clearly evident in non-profit organizations such as research and development institutions, and educational institutions. Top management wants these organizational goals to be attained, out other participants have their own personal goals that they want to achieve. These personal goals are the satisfaction of their needs. In other words, participants act in their own self- interest. Here individuals may grow bigger than the organization and this may lead to goal conflict. The control system should be designed so as to integrate the personal goals with organizational goals, and thereby achieve goal congruence. As managers tend to take action according to their perceived self-interest, the control system should ensure that these actions are also in the interest of the organization. Thus, the system should discourage

16

individuals acting against the interests of the organization, e.g., a cost reduction should not be achieved at the cost of quality if the organization has concern for quality products. In the language of social psychology, the management control system should encourage goal congruence; that is, it should be structured so that the goals of participants, so far as is feasible, are consistent with the goals of the organization as a whole. If this situation exists, a decision that a manager regards as being good from his own viewpoint will also be good decision for the organization as a whole. As McGregor states, The essential task of management is to arrange organizational conditions and methods of operations so that people can achieve their own goals best by directing their own efforts towards organizational objectives. Informal Factors that influence goal congruence consists of external factors and internal:

External factors, namely the norms of behavior expected to occur in the community (and the organization is part of the community).

External factors affecting goal congruence is the work ethic and appropriate industryspecific norms.

Working ethos is one of the organization's loyalty, and perseverance, spirit and pride that have in carrying out their duties.

The internal factors that affect goal congruence is the culture, management style, informal relationships within the organization and perception and communication.

Culture includes beliefs in the organization together, the values embraced life, behavioral norms and assumptions that implicitly and explicitly accepted applied at all levels of the organization.

Culture is strongly influenced by the personality and policy managers. Style of management has the strongest impact on management control, because the attitude is a reflection of his subordinates superiors.

Informal relations are also needed, although the formal relationship has been established.

The means to achieve organizational goals must also be well communicated and the messages conveyed are expected to be interpreted with the same meaning.

17

Q-9 Write Short Notes on the following: 1) Zero Based Budgeting Ans:Zero-based budgeting is an approach to planning and decision-making which reverses the working process of traditional budgeting. In traditional incremental budgeting (Historic Budgeting), departmental managers justify only variances versus past years, based on the assumption that the "baseline" is automatically approved. By contrast, in zero-based budgeting, every line item of the budget must be approved, rather than only changes. During the review process, no reference is made to the previous level of expenditure. Zero-based budgeting requires the budget request be re-evaluated thoroughly, starting from the zero-base. This process is independent of whether the total budget or specific line items are increasing or decreasing. According to Sarant, Zero Based Budgeting is a technique which complements and links to existing planning, budgeting and review processes. It identifies alternative and efficient methods of utilizing limited resources. It is a flexible management approach which provides a credible rationale for reallocating resources by focusing on a systematic review and justification of the funding and performance levels of current programs. Following are the Advantages and Disadvantages of Zero Based Budgeting: ADVANTAGES: 1. Efficient allocation of resources, as it is based on needs and benefits rather than history. 2. Drives managers to find cost effective ways to improve operations. 3. Detects inflated budgets. 4. Increases staff motivation by providing greater initiative and responsibility in decisionmaking. 5. Increases communication and coordination within the organization. 6. Identifies and eliminates wasteful and obsolete operations. 7. Identifies opportunities for outsourcing. 8. Forces cost centers to identify their mission and their relationship to overall goals.
18

9. Helps in identifying areas of wasteful expenditure, and if desired, can also be used for suggesting alternative courses of action. DISADVANTAGES: 1. More time-consuming than incremental budgeting. 2. Justifying every line item can be problematic for departments with intangible outputs. 3. Requires specific training, due to increased complexity vs. incremental budgeting. 4. In a large organization, the amount of information backing up the budgeting process may be overwhelming. Q9. 2) Free Cash Flow Ans:Free Cash Flow is a measure of financial performance calculated as operating cash flow minus capital expenditures. Free cash flow (FCF) represents the cash that a company is able to generate after laying out the money required to maintain or expand its asset base. Free cash flow is important because it allows a company to pursue opportunities that enhance shareholder value. Without cash, it's tough to develop new products, make acquisitions, pay dividends and reduce debt. FCF is calculated as:

It can also be calculated by taking operating cash flow and subtracting capital expenditures. Free Cash Flow of the Firm is calculated as follows: A measure of financial performance that expresses the net amount of cash that is generated for the firm, consisting of expenses, taxes and changes in net working capital and investments. Calculated as:

19

This is a measurement of a company's profitability after all expenses and reinvestments. It's one of the many benchmarks used to compare and analyse financial health. A positive value would indicate that the firm has cash left after expenses. A negative value, on the other hand, would indicate that the firm has not generated enough revenue to cover its costs and investment activities. In that instance, an investor should dig deeper to assess why this is happening - it could be a sign that the company may have some deeper problems. Q9. 3) MCS in the Matrix Organization Ans:The matrix organization is an organizational structure in which the work is divided in projects. Each project is a profit center and is looked after by the project manager. Each project team has functional level employees that report to their respective functional managers and their project manager. Matrix Organizational structure assigns multiple responsibilities to the functional heads. Evaluation of performance of such organizational entities is very difficult. It poses the problem of casting individual responsibility. This form of organization is very complex, from the point of view of management control system. Usually in an advertisement agency, account supervisors are shifted from one account to another on periodic basis. This practice allows the agency to look at the account from the perspectives of different executives. However taking in to consideration the time lag of result, realization in such services is quite large. This may pose problem of performance assessment of a particular executive. This does not mean that a control system designer should insist on abandoning the rotation system of the executives. Matrix structure offers advantages such as faster decision making process, efficiency and effectiveness. But simultaneously, it may pose problems such as added complexity in control function, assignment of responsibility and authority etc. Following are the advantages and disadvantages of Matrix organizational structure: ADVANTAGES: 1. Minimization of project costs, due to sharing of resources. 2. Minimization of conflicts and Stress distribution between the teams.

20

3. Balance between time, cost and performance. 4. Sharing of authority and responsibility. 5. Information sharing. 6. Resource sharing. 7. Ideal for project based organization. 8. Better coordination between the team. DISADVANTAGES: 1. Two bosses can create conflict of authority. 2. Limited applicability. 3. Not suitable for small organizations. 4. Complete responsibility of the manager for success or failure. 5. Suitable only for project based organizations. At the end, we must not forget that the management control system is for the organization and not the organization exists for management control system. One has to mold and remold the management control system to suit the given organization structure.

21

UNIVERSITY PAPER SOLUTION YEAR- 2010

22

Q1.Framework of Management control Ans: Management control is the process by which managers influence other members of the organization to implement the organizations strategies. Management control process involves informal interactions between one manger and another manager and his or her subordinates. Informal communications occurs by means of memoranda, meetings, conversations, and even by facial expression. The informal interactions take place within a formal planning and control system. Such system includes the following activities: 1] Strategic planning, 2] Budget preparation, 3] Execution, 4] Evaluation of performance. Each activity leads to next in a regular cycle. 1] Strategic planning: it is process of deciding on the major programs that organization undertakes to implement its strategies and appropriate amount of resources that will be devoted to each. The output of the process called as strategic planning. This is the first step in management control cycle. 2] Budget planning: budget represent fine tuning of the strategic planning, incorporating most current information. In budget, revenue and expenses are rearranged from programme to the responsibility centre, thus budget shows the expenses that each managers expected to occur. The process of budget preparation is essentially one of the negotiations between the managers of each responsibility centre and their superior. 3] Execution: managers execute the programme or part of the programme for which they are responsible and also report on what has happened in the course of fulfilling that responsibility. Reports on responsibility centre may show budgeted and actual information, financial and non-financial performance measures, internal & external information. 4] Evaluation of performance: the process of evaluation is comparison of actual expenses and those that should have been incurred under circumstances. If the circumstances assumed in the budget process are unchanged, the comparison between budgeted and actual amounts. If circumstances have changed, these changes are taken into accounts. Ultimately, the analysis leads to praise or constructive criticism of the responsibility centre managers. Where management control is imposed, it functions within the framework established by the strategy. Normally these objectives (standards) are established for major subsystems within the organization, such as SBUs, projects, products, functions, and responsibility centers. Typical management control measures include ROI, residual income, cost, product quality, and so on. These control measures are essentially summations of operational control measures. Corrective action may involve very minor or very major changes in the strategy. Operational control systems are designed to ensure that day-to-day actions are consistent with established plans and objectives. It focuses on events in a recent period. Operational control systems are derived from the requirements of the management control system.
23

Corrective action is taken where performance does not meet standards. This action may involve training, motivation, leadership, discipline, or termination. The differences between strategic and operational control are highlighted by reference to a general definition of management control: "Management control is the set of measurement, analysis, and action decisions required for the timely management of the continuing operation of a process". "Strategic planning is the process of deciding on the goals of the organization and the strategies for attaining these goals." Strategies are guidelines for deciding the appropriate actions for attaining the organization's goals. The essential difference between strategic planning and management control is that the strategic planning process is unsystematic. Strategic control occurs in two ways. First, strategic planning is itself a form of control. Second, strategic plans are converted into reality not only by their influence on the management control activity but also by the key decisions regarding allocation of resources. Second, while capital budgeting systems can respond to requests for resources that are consistent with the accepted strategic plan, the period between formal, comprehensive strategic planning exercises can give rise to unanticipated changes in the environment or unexpected internal crises. Q2. Expense centers are responsibility centers whose inputs are measured in monetary terms whose output are not. There are two general types of expenses centers: - engineered and discretionary. These labels relates to two types of cost. Engineered costs are those for which the right or proper amount can be estimated with reliability. For example, factorys costs for direct labour, direct material, components supplier and utilities. Discretionary costs are those for which no such engineered estimate is feasible. In discretionary expenses centers, the cost incurred depends on managements judgment as to the appropriate amount under the circumstances. 1. 2. 3. 4. Engineered expenses centers:- it have following characteristics:The profit input can be measured in monetary terms. Their output can be measured in physical terms. The optimum dollars amount of input required to produce one output can be determined.

Diagram:Optimal relationship can be established Input Output

Work
(Dollar) (Physical)

Manufacture function

24

Engineered expense centers Engineered expense centers are usually found in manufacturing operation, warehouse distribution and similar units within the marketing organization may also be engineered expenses centers as certain responsibility centers within administrative and support departments for instance, account receivable, account payable and pay roll sections in controller department. Such unit performs repetitive task for which standard cost can be developed. These centers are usually located within departments that are discretionary expenses centers. In engineered expenses centers output are multiplied by standard cost of each unit produce measured what the finished products should have cost. Managers of engineered expenses center may be responsible for activities such as training and employee development that are not related to current production. The term engineered expense center refers to responsibility centers in which engineered cost predominate, but it does not imply that valid engineered estimates can be made for each and every cost items. Discretionary expenses centers:Discretionary expenses centers include administrative and supports units, research and developments operation and most marketing activities. The output of these centers can not be measured in monetary terms. Diagram:Optimal relationship cannot be established Input Output

Work
(Dollar) (Physical)

R&D function

The term discretionary does not imply that managements judgment to optimum cost is capricious rather it reflects managements decision regarding certain policies, whether to match the marketing effort of competitors; the level of service the company should provide to its customer and appropriate amount to spend for R&D, public relations and other activities. One company may have a similar small head quarters staff, while another company of similar size and same industry may have staff 10 times as large. The senior manager of each company may each to be convinced that their respective decision on staff size are correct but their is no objective to judge which is right; both decision may be equally good under the circumstances with the differences in the two companies. In discretionary centers, the differences between budget and actual expenses are not a measure of efficiency. Rather it is simply the differences between the budgeted input and actual does not incorporate the value of output if actual expenses do not exceed the budget amount, the manager has lived within the budget, but since by definition the budget does not
25

to predict the optimum amount of spending living within the budget does not necessarily indicate efficient performance. Budget preparation and performance evaluation Managements make budgetary decision for discretionary expenses centers that differ from those for engineered expenses centers. For the later, it decides whether the proposed operating budget represents the unit of performance efficiently. Its volume is not a major concern; this is largely determined by the action of other responsibility centers. The marketing departments ability to generate sales. This work done by discretionary expenses centers falls into two category; continuing and special. Continuing work is done consistently from year to year, such as preparation of financial statement by controller office. Special work is a one shot project for example developing and installing a profit budgeting system in new ly acquired division as per the discretionary expenses center budget is a management by objective, a formal process in which budgeter process to accomplish specific job. Incremental budgeting in this model discretionary expenses centers current level of expenses is taken as starting point. This amount is adjusted foe inflation, anticipated change in the workload of continuing job. Incremental budgeting has two characteristics and two drawbacks. The drawbacks are first, the discretionary expenses centers current level of expenditure is accepted and not reexamined during the budget preparation process and second, manager of these center typically want to increase the level of services and tend to request additional resources, which they make a sufficiently strong case are usually provide. Zero base review An alternative budgeting approach is to make through analysis of each discretionary expenses center on rolling schedule, so that all are reviewed at least once every five years or so. Such an analysis is often called zero base review. In contrast with incremental budgeting this alternative intensive review attempts to ascertain, devolve that is form scratch, the resource actually required to carry out each activity within the expenses centers. This analysis established another new base, at which point the annual budget review simply attempts to keep costs reasonably in line with this base until the next review take place, five year down line. Zero base review is time consuming and they are likely to be traumatic for the managers whose operations are being reviewed. Also managers will not only do their best to justify their current level of spending, but also attempts to thwart the effort, regarding zero base review as something to be put off indefinitely in favour of more pressing business Cost variability Unlike cost engineered expenses center, which are strongly affected by short run volume change costs in discretionary expenses center are comparatively insulated from such short term fluctuation. This differences stems from the facts that in preparing the budget for discretionary expenses center. Management tends to approve change that correspond anticipated change in sales. Type of financial control Financial control in a discretionary expenses center is quite different from than the engineered expenses center. The objective become cost competitive by selling a standard
26

actual cost against this standards. The main purpose is to control the cost by allowing manager to participate in planning. Measurement of performance The primary job of discretionary expenses centers manager is to obtain the desired output spending and an amount that is on budget to do this satisfactory. Spending more than that is cause for concern and spending less may indicate that planned work is wont be done. In discretionary centers as apposed to engineered expense centers, the financial performance report is not a means of evaluating the efficiency of manager. Control over spending can be exercised by required the supervisors approval before the budget is overrun is permitted without additional approval. Q3. Q1.Zero Based Budgeting It is an approach to planning and decision-making which reverses the working process of traditional budgeting. In traditional incremental budgeting (Historic Budgeting), departmental managers justify only variances versus past years, based on the assumption that the "baseline" is automatically approved. By contrast, in zero-based budgeting, every line item of the budget must be approved, rather than only changes.During the review process, no reference is made to the previous level of expenditure. Zero-based budgeting requires the budget request be reevaluated thoroughly, starting from the zero-base. This process is independent of whether the total budget or specific line items are increasing or decreasing.The term "zero-based budgeting" is sometimes used in personal finance to describe "zero-sum budgeting", the practice of budgeting every dollar of income received, and then adjusting some part of the budget downward for every other part that needs to be adjusted upward. Zero based budgeting also refers to the identification of a task or tasks and then funding resources to complete the task independent of current resourcing.A method of budgeting in which all expenses must be justified for each new period. Zero-based budgeting starts from a "zero base" and every function within an organization is analyzed for its needs and costs. Budgets are then built around what is needed for the upcoming period, regardless of whether the budget is higher or lower than the previous one. ZBB allows top-level strategic goals to be implemented into the budgeting process by tying them to specific functional areas of the organization, where costs can be first grouped, then measured against previous results and current expectations. Because of its detail-oriented nature, zero-based budgeting may be a rolling process done over several years, with only a few functional areas reviewed at a time by managers or group leadership. Zero-based budgeting can lower costs by avoiding blanket increases or decreases to a prior period's budget. It is, however, a time-consuming process that takes much longer than traditional, cost-based budgeting. The practice also favors areas that achieve direct revenues or production; their contributions are more easily justified than in departments such as client service and research and development. 2.FREE CASH FLOW

27

A measure of financial performance calculated as operating cash flow minus capital expenditures. Free cash flow (FCF) represents the cash that a company is able to generate after laying out the money required to maintain or expand its asset base. Free cash flow is important because it allows a company to pursue opportunities that enhance shareholder value. Without cash, it's tough to develop new products, make acquisitions, pay dividends and reduce debt. FCF is calculated as:

It can also be calculated by taking operating cash flow and subtracting capital expenditures. Free Cash Flow of the Firm is calculated as follows:A measure of financial performance that expresses the net amount of cash that is generated for the firm, consisting of expenses, taxes and changes in net working capital and investments. Calculated as:

This is a measurement of a company's profitability after all expenses and reinvestments. It's one of the many benchmarks used to compare and analyze financialhealth. A positive value would indicate that the firm has cash left after expenses. A negative value, on the other hand, would indicate that the firm has not generated enough revenue to cover its costs and investment activities. In that instance, an investor should dig deeper to assess why this is happening - it could be a sign that the company may have some deeper problems. Q.4 Definition . An autonomous division or organizational unit, small enough to be flexible and large enough to exercise control over most of the factors affecting its long-term performance. Because strategic business units are more agile (and usually have independent missions and objectives), they allow the owning conglomerate to respond quickly to changing economic or market situations. In other words, In business, a strategic business unit (SBU) is a profit center which focuses on product offering and market segment. SBUs typically have a discrete marketing plan, analysis of competition, and marketing campaign, even though they may be part of a larger business entity. An SBU may be a business unit within a larger corporation, or it may be a business unto itself. Corporations may be composed of multiple SBUs, each of which is responsible for its own profitability.
28

Feature of SBUs The main features are:


Location in the structure of the company, These are organizational units without legal personality, They utilize formula "product-market", Type of activity performed by them is of crucial and decisive importance for the whole company, Functional and decision-making autonomy include: laboratory testing, production preparation, production, finance, accounting and marketing, SBU has divisionalized structure, which is determined by the size of production, technology and research activities, financial and accounting processes and marketing influences.

Conditions for creating SBUs The main philosophical concept behind the formation of strategic business units is to serve a clear and defined market segment along with a clear and defined strategy. These business units have to contain all the needs and corporate capabilities of the respective organization. The entire portfolio of the concerned business has to be managed by allocation of managerial and capital resources for serving the overall interest of the entire organization. This helps in developing a balance in the earnings, sales and the assets at a level which is controlled and acceptable for taking the right amount of risks. The strategic business unit (SBU) is created with the application of set criteria which consist of the competitors, price models, customer groups and the overall experience of the company. It is also sometimes seen that a number of different verticals present in the same organization having similar competitors and target customers are amalgamated to form a single SBU. This helps in strategically planning the overall business of the organization. This is also true for the company which has different product ranges and some of them have similar capabilities in terms of research and development, marketing and manufacturing. Such products can also be amalgamated to form a single unit. The main notion which rests behind the concept of strategic business units is to gain a competitive advantage in the populated marketplace. This can be done because the SBU helps in segmenting the activities of the company in a strategic manner and the resources are thus allocated competitively. Each Business Unit must meet the following criteria: 1.Have a unique business mission, independent from other SBUs. 2.Have clearly definable set of competitors. 3.Be able to carry out integrative planning relatively independently of other SBUs. 4.Should have a Manager authorized and responsible for its operation. 5.Be large enough to justify senior management attention but small enough to serve as auseful focus for resource allocation.

29

SBUs advantages. The main advantages are:


SBU supports cooperation between the departments of the company which has a similar range of activities; Improvement of strategic management Improvement of accounting operations, Easier planning of activities

SBUs disadvantages.

Difficulty with contact with higher management May cause of internal tension due to difficult access to internal and external sources of funding, May be the cause of the unclear situation with regard to the management activities.

Measures of judging performance of SBUs EVA Balance score cardS Q5. Transfer pricing refer to the amount used in accounting for any transfer of goods and services between responsibility centers. This is what a narrow definition and limit the term transfer price to the value placed on a transfer of goods or services in transaction in which at least one of the two parties involved in the profit center. Such a price typically includes a profit element because an independent company normally would not transfer goods or services to another independent company at cost or less. Therefore, the mechanism for allocating cost in an accounting system; such cost do not include a profit element. The term price as used here has the same meaning as it has when used in connection transaction between independent companies. Objective of transfer pricing Profit center are responsible for product development, manufacturing and marketing each share in the revenue generated when the product is finally sold. The transfer price should be designed so that it accomplishes the following objectives: 1. 2. 3. 4. it should provide unit with the relevant information it needs to determine the optimum trade-off between company cost and revenues It should induce goal congruence decision i.e., the system should be designed so that decision that improve business unit profit will also improve company profits. It should help to measure the economic performance of the individual business units. The system should be simple to understand and easy to administer.
30

Thus, from the objective, it is understandable that the Transfer price is mainly transferring of goods and services from one unit to another where much important is not given to accounting basis but also to all other effect. Different Methods To Arrive at Transfer Base Price 1. MARKET BASED PRICES:Market price refers to a price in an intermediate market between independent buyer and seller. When there is competitive external market for the transferred product, market prices work well as transfer prices. When transferred goods are recorded at market prices divisional performance is more likely to represent the real economic contribution of the division to total company profit. If the goods cannot be brought from a division within the company the intermediate product would have to be purchased from the current market price from the outside market. Divisional profits are therefore likely to be similar to the profits that would be calculated if the divisions were to be separate organizations. Divisional profits can be compared directly with the profitability of similar companies operating in the same type of business. No divisions can benefit at the expense of another division. The selling division can sell all that it produces at the market price transferring internally at a lower price would make that division worse off. Since the minimum transfer price for the selling division is the market price and the maximum price for buying division is also the market price the only possible transfer price is the market price. The market price can be used to resolve conflicts among the buying and selling division. Market price is optimal so long as the selling division is operating at full capacity. Market price may change often. Internal selling expense may be less than would be incurred if the products were sold to outside. 2. COST BASED PRICES:When external markets do not exist or are not available to the company or when the information about external market prices is not readily available companies may decide to use some form of cost based transfer pricing system. Under this method the transfer price is based on the total product cost. It has three characteristics. 1. It provides a varying price since cost per unit keeps changing as use of capacity changes. 2. It mixes short run and long run costs.
31

3. The concept is based on the equation of variable cost plus arbitrary mark up to cover capacity related cost and a targeted profit margin. 3.Negotiated Transfer Prices: Negotiated transfer pricing has the advantage of emulating a free market in which divisional managers buy and sell from each other in a manner that simulates arms-length transactions. However, there is no reason to assume that the outcome of these transfer price negotiations will serve the best interests of the company or shareholders. The transfer price could depend on which divisional manager is the better poker player, rather than whether the transfer results in profit-maximizing production and sourcing decisions. Also, if divisional managers fail to reach an agreement on price, even though the transfer is in the best interests of the company, senior management might decide to impose a transfer price. However, senior managements imposition of a transfer price defeats the motivation for using a negotiated transfer price in the first place. Q6. Definition of 'Profit Center' A branch or division of a company that is accounted for on a standalone basis for the purposes of profit calculation. A profit center is responsible for generating its own results and earnings, and as such, its managers generally have decision-making authority related to product pricing and operating expenses. Profit centers are crucial in determining which units are the most and least profitable within an organization. The concept of profit centers enables a company's executives and management to determine how best to focus its resources more resources to highly profitable areas, while reducing allocations to less profitable or loss-making units. Not all units within an organization can be tracked as profit centers. This is especially applicable to departments that provide an essential service within an organization, but do not generate their own revenues. Some examples of these include the research department within a broker-dealer, the administration arm of a company, and a unit that provides after-sales support in an organization. Definition of 'Investment Center' A business unit that can utilize capital to directly contribute to a company's profitability. Companies evaluate the performance of an investment center according to the revenues it brings in through investments in capital assets compared to the overall expenses. An investment center is sometimes called an investment division. An investment center is different than a cost center, which indirectly adds profit and is evaluated according to the money it takes to operate. Moreover, unlike a profit center, investment centers can utilize capital in order to purchase other assets. Because of this complexity, companies have to use a variety of metrics, including return on investment (ROI), residual income and economic value added (EVA) to evaluate performance.

32

Difference between investment center & profit center a. an investment center incurs costs, but does not directly generate revenues. b. an investment center incurs no costs but does generate revenues. c. an investment center is responsible for effectively using center assets. d. an investment center provides services to profit centers e. There is no difference; investment center and profit center are synonymous. Measures of judging performance of profit centers. Types of Profitability Measures A profit center's economic performance is always measured by net income (i.e., the income remaining after all costs, including a fair share of the corporate overhead, have been allocated to the profit center). The performance of the profit center manager, however, may be evaluated by five different measures of profitability: (1) contribution margin, (2) direct profit, (3) controllable profit, (4) income before income taxes, or (5) net income (1) Contribution Margin: Contribution margin reflects the spread between revenue and variable expenses. The principal argument in favor of using it to measure the performance of profit center managers is that since fixed expenses are beyond their control, managers should focus their attention on maximizing contribution. The problem with this argument is that its premises are inaccurate; in fact, almost all fixed expenses are at least partially controllable by the manager, and some are entirely controllable. Many expense items are discretionary; that is, they can be changed at the discretion of the profit center manager. Presumably, senior management wants the profit center to keep these discretionary expenses in line with amounts agreed on in the budgetfo rm ul at i on pr oc es s. A f o cus o n t h e co nt ri b ut i on m ar g i n t en ds t o di r e ct at t e nt i o n a w a y f rom t h i s responsibility. Further, even if an expense, such as administrative salaries, cannot be changed in the short run, the profit center manager is still responsible for controlling employees' efficiency and productivity. (2) Direct Profit: This measure reflects a profit center's contribution to the general overhead and profit of the corporation. It incorporates all expenses either incurred by or directly traceable to the profit center, regardless of whether or not these items are within the profit center manager's control. Expenses incurred at headquarters, however, are not included in this calculation. A weakness of the direct profit measure is that it does not recognize the motivational benefit of charging headquarters costs. (3) Controllable Profit: Headquarters expenses can be divided into two categories: controllable and non controllable. The former category includes expenses that are controllable, at least to a degree, by the business unit manager-information technology services, for example. If these costs are included in the measurement system, profit wi l l be w h at r e m ai ns a ft e r t he de du ct i on o f al l ex p ens e s t ha t m a y b e i n fl u en c ed b y t h e pr ofi t c ent e r manager. A major disadvantage of this measure is that because it excludes non controllable headquarters expenses it cannot be directly compared with either published data or trade association data reporting the profits of other companies in the industry. (4) Income before Taxes

33

n this measure, all corporate overhead is allocated to profit centers based on the relative amount of expense each profit center incurs. There are two arguments against such allocations. First, since the costs incurred by corporate staff departments such as finance, accounting, and human resource management are not controllable by profit center managers, these managers should not be held accountable for them. Second, it may be difficult to allocate corporate staff services in a manner that would properly reflect the amount of costs incurred by each profit center 5) Net Income: Here, companies measure the performance of domestic profit centers according to the bottom line, the amount of net income after income tax. There are two principal arguments against using this measure: (1)after tax income is often a constant percentage of the pretax income, in which case there would be no advantage in incorporating income taxes, and (2) since many of the decisions that affect income taxes are made at headquarters, it is not appropriate to judge profit center managers on the consequences of these decisions. There are situations, however, in which the effective income tax rate does vary among profit centers. For example, foreign subsidiaries or business units with foreign operations may have differente ff e ct i v e i n com e t ax r at es . In o t he r c a se s, p ro fi t c e nt e rs m a y i n fl u en c e i n com e t ax es t hr ou gh t h ei r installment credit policies, their decisions on acquiring or disposing of equipment, and their use of other generally accepted accounting procedures to distinguish gross income from taxable income. In these situations, it may be desirable to allocate income tax expenses. Measures of judging performance of investment centers. profit percentage (profit divided by sales), return on investment (profit divided by initial investment), residual income (profit minus a deduction for capital costs). Profits are seldom a viable measure at the cost center level, however. Rather, performance is most often measured by comparing actual costs against a budget. A variance is defined as the difference between the amount budgeted for a particular activity and the actual cost of carrying out that activity during a given period. Variances may be positive (under budget) or negative (over budget). Performance data can be developed for management purposes independent of the budget and control accounts. This kind of performance reporting has been used in the justification of resource requests and in the assessment of cost and work progress where activities are fairly routine and repetitive. Under this approach, units of work are identified, and changes in quantity (and, on occasion, quality) of such units are measured as a basis for analyzing financial requirements. The impact of various levels of service can be tested, and an assessment can be made of changes in the size of the client groups to be served. This approach is built on the assumption that certain fixed costs remain fairly constant regardless of the level of service provided and that certain variable costs change with the level of service or the size of the clientele group served. Marginal costs for each additional increment of service provided can be determined through such an approach. With the application of appropriate budgetary guide-lines, these costs can then be converted into total cost estimates.
34

Variances, budgeted results, and other techniques of responsibility accounting are relatively neutral devices. When viewed positively, they can provide managers with significant means of improving future decisions. They can also assist in the delegation of decision responsibility to lower levels within an organization. These techniques, however, are frequently misused as negative management tools--as means of finding fault or placing blame. This negative use stems, in large part, from a misunderstanding of the rationale of responsibility accounting. Passing the buck is an all-too-pervasive tendency in many large organizations. This tendency is supposedly minimized, however, when responsibility is firmly fixed. Nevertheless, a delicate balance must be maintained between the careful delineation of responsibility, on the one hand, and an overly rigid separation of responsibility, on the other. Many activities may fall between the cracks when responsibility is too strictly prescribed. This problem is particularly evident when two or more activities are interdependent. Under such circumstances, responsibility cannot be delegated too far down in the organization, but must be maintained at a level that will ensure cooperation among the units that must interact if the activities are to be carried out successfully

Q7. The Balanced Scorecard (BSC) is a performance management tool which began as a concept for measuring whether the smaller-scale operational activities of a company are aligned with its larger-scale objectives in terms of vision and strategy.By focusing not only on financial outcomes but also on the operational, marketing and developmental inputs to these, the Balanced Scorecard helps provide a more comprehensive view of a business, which in turn helps organizations act in their best long-term interests. The balanced scorecard approach provides a clear prescription as to what companies should measure in order to 'balance' the financial perspective. The balanced scorecard is a management system (not only a measurement system) that enables organizations to clarify their vision and strategy and translate them into action. It provides feedback around both the internal business processes and external outcomes in order to continuously improve strategic performance and results. It captures both the financial and non-financial aspects of a company's strategy and discusses the cause and effect relationship that drives business results. It is a proven tool to translate a company's strategy into action."The balanced scorecard retains traditional financial measures. But financial measures tell the story of past events, an adequate story for industrial age companies for which investments in long-term capabilities and customer relationships were not critical for success. These financial measures are inadequate, however, for guiding and evaluating the journey that information age
35

companies must make to create future value through investment in customers, suppliers, employees, processes, technology, and innovation."The balanced scorecard suggests that we view the organization from four perspectives, and develop metrics, collect data and analyze it relative to each of these perspectives:

1.

The Learning and Growth Perspective This perspective includes employee training and corporate cultural attitudes related to

both individual and corporate self-improvement. In a knowledge economy, Human resources are the main resource. In the current climate of rapid technological change, it is becoming necessary for knowledge workers to be in a continuous learning mode. The Organisation has to identify the infrastructure that must be built in order to create long term growth and improvement. The objective is to build up mechanism to fill up the existing gaps in knowledge and processes and to be continually innovative. Kaplan and Norton emphasize that 'learning' is more than 'training'; it also includes things like mentors and tutors within the organization, as well as that ease of communication among workers that allows them to readily get help on a problem when it is needed. 2. The Business Process Perspective

This perspective refers to internal business processes. This perspective allows the managers to know how well their business is running, and whether its products and services conform to customer requirements. It should be carefully analyzed by those who know these processes most intimately. The main focus of this perspective is on defining the critical business processes in which the organization seeks to excel. The company must understand its value chain. In addition to the strategic management process, two kinds of business processes may be 3. identified: a) mission-oriented processes, and b) support

The Customer Perspective

Current business philosophy has shown an increasing realization of the importance of customer focus and customer satisfaction in any business. These are leading indicators: if customers are not satisfied, they will eventually find other suppliers that will meet their needs. Poor performance from this perspective is thus a leading indicator of future decline, even though the current financial picture may look good. Key parameters in this perspective to be analyzed are customer satisfaction, customer retention, new customer acquisition and

36

market share in targeted segments. Major Dimension's affecting customer response and profitability are product and service attributes, customer relations and image and reputation. 4. The Financial Perspective

These measures do not disregard the traditional need for financial data. Timely and accurate financial data will always be a priority, and managers will do whatever necessary to provide it. In fact, often there is more than enough handling and processing of financial data. With the implementation of a corporate database, it is hoped that more of the processing can be centralized and automated. But the point is that the current emphasis on financials leads to the "unbalanced" situation with regard to other perspectives. A Balanced Scorecard must be balanced. In this context, the internal auditor may prove helpful to create and monitor the scorecard since he has access to company wide measure of performance. Performance measurement system: additional consideration A performance measurement system attempt to address the need of the different stakeholders of the organization by crating a blend of strategic measures: outcome and driver measures, financial and no financial measures and internal & external measures. Outcome and driver measures Outcome measurements indicate the result of strategy(e.g. increased revenue). These measures typically are lagging indicators; they tell management what has happened. By contrast driver measures are leading indicators they show the progress of key areas in implementing a strategy. Whereas outcome measures indicate only the final result driver measures can be used at a lower level and indicate incremental changes that will ultimately affect the outcome.By focusing management attention on key aspect of the business driver measures affect behavior in the organisation. If a business units desire is to improve time to market focusing on cycle time allows management to track how well this goal is being achieved, which in turn encourages employees to improve this particular measures. Outcome and driver measures are inextricably linked. if outcome measures indicate there is a problem but the driver measures indicate the strategy is being implemented well there is a high chance that the strategy needs to be changed. Financial and no financial measures

37

Organization has developed very sophisticated system to measure financial performance. Unfortunately as many U.S..Firm discovered, during the 1980s industries were being driven by changes in nonfinanacial areas, such as quality and customer satisfaction, that eventually impacted companies financial performance.Even through they recognize the importance of nonfinanacial measures, many organizations have failed to incorporate them into their executive level performance reviews because these measures tend to be much less sophisticated than financial measures and senior management is less adept at using them. Internal and External measures Companies must strike a balance between external measures such as customer satisfaction and measures of internal business processes, such as manufacturing yield. Too often companies sacrifice internal development for external results altogether, mistakenly believing that good internal measures are sufficient. Measurements drive change The most important aspect of the performance measurement system is its ability to measure outcome and diverse in a way that causes the organisation to act in accordance with its strategies. The organisation achieve goal congruence by linking overall financial and strategic objective with lower levels. With these measures all employee can understand how their action impact the company strategies.Because these measures are explicitly tied to an organizations strategies, the measures in the scorecard must be strategy specific and therefore organization specific and therefore organization-specific. While a generic performance measurement framework exists, there is no such thing a generic scorecard.The scorecard measures are linked from top to bottom and tied to specific targets throughout the entire organization. Objectives can further clarify a strategy so that the organization knows both what it needs to do how much.Finally the scorecard emphasis the idea of cause and effect relationship among measures. By explicitly presenting the cause and effect relationships among measures. by explicitly presenting the cause and effect relationship, an organization will understand how nonfinanacial measures(e.g. product quality) drive financial measures (e.g. revenue) bellow figure present an example of how the measures link to each other in a cause and effect relationship. Better selection, training, and development of manufacturing employees (measured in term of manufacturing yield) lead to better product quality(measured in term of first pass yield) and better on time delivery (measured in term

38

of order cycle time). These improvements in turn lead to improved customer loyalty which leads to enhanced sales.(measured in term of sales growth) The scorecard must not simply be a laundry list of measures. Rather, the individual measures in the scorecard must be linked together explicitly in a cause effect way, as a tool to translate strategy into action. Perspective
Innovation and learning perspective

Measures
Manufacturing skills

Internal business perspective

First pass yield order cycle time

Customer perspective

Customer satisfaction survey.

Financial perspective

Sales revenue growth

Therefore balance scorecard is greater measure than performance appraisal. Example Balanced Scorecard: Regional Airline Mission: Dedication to the highest quality of Customer Service delivered with a sense of warmth, friendliness, individual pride, and Company Spirit. Vision: Continue building on our unique position -- the only short haul, low-fare, high-frequency, point-to-point carrier in America.z

39

Theme: Efficiency

Operating Objectives

Measures

Targets

Initiatives

Financial Profitability Profitability Fewer planes Lower costs Increased revenue Increase Revenue Market Value Seat Revenue Plane Lease cost 25% per year 20% per year 5% per year Optimize routes Standardize plans

Customer Flight is on - FAA time Time Lowest prices More Customers More Customers Arrival Rating Customer Ranking No. Customers On First in industry 98% Satisfaction % change Quality management Customer loyalty program

On-time flights

Lowest Prices

40

Internal Fast ground Improve Turnaround Time turnaround On Ground 25 Time Minutes On-Time 93% Departure Cycle time optimization program

Learning

Ground crew alignment

Align Ground Crews

% Ground yr. 1 70% crew yr. 4 90% stockholders yr. 6 100% % Ground crew trained

Stock ownership plan Ground crew training

Q.8 Service organization A service club or service organization is a voluntary non-profit organization where members meet regularly to perform charitable works either by direct hands-on efforts or by raising money for other organizations. A service club is defined first by its service mission. Second its membership benefits, such as social occasions, networking, and personal growth opportunities encourage involvement A service organization is not necessarily exclusive of ideological motives, although organizations with such defined motives are more likely to identify themselves through their association. Much like the historical religious organizations formed the basis for many of societal institutions, such as hospitals, service organizations perform many essential services for their community and other worthy causes. In the United States, some of these clubs usually also have a component club organization that is a tax exempt. Manufacturing organization manufacturers integrate all elements of the manufacturing system to satisfy the needs and wants of its customers in a timely and effective manner. They eliminate organizational barriers to permit improved communication and to provide high quality products and services.

41

manufacturing is a wealth-producing sector of an economy, whereas a service sector tends to be wealth-consuming. Emerging technologies have provided some new growth in advanced manufacturing employment opportunities in the Manufacturing Belt in the United States. Manufacturing provides important material support for national infrastructure and for national defense. Difference between professional service organization & normal service organization. Professional service organization : Professional services is an industry of technical or unique functions performed by independent contractors or consultants whose occupation is the rendering of such services. Examples of professional services include those of: accountants, actuaries, appraisers, architects, attorneys, business consultants, business development managers, copywriters, engineers, funeral directors, law firms, public relations professionals, recruiters, researchers, real estate brokers, translators and medical centers. While not limited to licentiates (individuals holding professional licenses), the services are considered "professional" and the contract may run to partnerships, firms, or corporations as well as to individuals. The difference between professional service organization & normal service organization can be the professional service organization provides expert services to their clients. Such as lawyers, advertising professionals, architects, accountants, financial advisers, engineers, and consultants, among others. Basically, they can be any organization or profession that offers customized, knowledge-based services to clients. Unlike other types of organizations, professional services firms sell knowledge and expertise not tangible, physical products. So these firms have different needs, and face different challenges. And in normal service organizations the services are provided voluntarily in the charitable form so that is the difference between normal & professional service organization. Pricing strategy for professional service organization Resource Allocation In the professional service rm, eectively managing human resources is the heart of the business. Selection, training and retention of world class service providers are critical. Then managing this team, whose members have an assortment of skills and experiences, across a national if not globalclient base, where each engagement is unique, is a daunting task. The successful professional services rm allocates people resources to those clients and activities that allow them to deliver superior, dierentiated value at competitive advantage. Service Delivery If putting the right people in the right places is the top priority, then delivering results that exceed expectations is the second. Successful professional services rms recognize from the get-go that they must build strong service histories to manage client perceptions, gain references and develop favorable word of mouth communication.
42

Again, this is a natural t for a value driven approach to pricing. Strong service histories enable the professional services rm to eectively demonstrate and document value delivered in monetary terms. This is the key to understanding the Economic Value Drivers of the sale. In sum, the greater the economic impact the more the customer can be convinced to pay for a service. Intellectual Property (IP) Management The competitive advantage of a professional services rm is built on the strength of their people; but where there are people, there is turnover, so a rm is at constant risk of losing its competitive advantage. The solution is protection and management of the rms intellectual property. Eective IP management permits the rm to grow capabilities over time, to get paid for meaningful dierentiation and to drive down costs of delivery. Value Exchange Ultimately, the most eective professional services pricing managers are masters at managing the value exchange. The value exchange is the continuing process whereby the customer is forced to acknowledge and pay for value delivery. And these words are used purposefully the customer is forced to acknowledge and pay for value delivery. Competitive Dynamics Of course, any discussion of pricing would be incomplete without reference to competition. However, experience from across many clients over many years has shown that most of the damage attributed to competitors is actually self inicted. Through unmanaged discounting, lack of eective sales incentives, weak value communication and negotiation strategies, and an unwillingness to charge for value delivered, many professional services rms have created the price sensitivity they see in their customers. Looking at it from the outside, it is patently obvious. From the inside, it looks like competition. Emotive Drivers Because of the intangible nature of professional services, purchase decisions often are made on the basis of a complex stream of decision variables. While some of these decision variables lend themselves to rigorous analysis, as in the Economic Value Drivers described above, other decision criteria are highly subjective. These are called the Emotive Drivers of the purchase, and they can be as important as the economic drivers to a professional services pricer.

43

Q.9 Corporate Level Strategy Corporate level strategy occupies the highest level of strategic decision-making and covers actions dealing with the objective of the firm, acquisition and allocation of resources and coordination of strategies of various SBUs for optimal performance. Top management of the organization makes such decisions. The nature of strategic decisions tends to be valueoriented, conceptual and less concrete than decisions at the business or functional level. Business-Level Strategy. Business-level strategy is applicable in those organizations, which have different businesses-and each business is treated as strategic business unit (SBU). The fundamental concept in SBU is to identify the discrete independent product/market segments served by an organization. Since each product/market segment has a distinct environment, a SBU is created for each such segment. For example, Reliance Industries Limited operates in textile fabrics, yarns, fibers, and a variety of petrochemical products. For each product group, the nature of market in terms of customers, competition, and marketing channel differs. There-fore, it requires different strategies for its different product groups. Thus, where SBU concept is applied, each SBU sets its own strategies to make the best use of its resources (its strategic advantages) given the environment it faces. At such a level, strategy is a comprehensive plan providing objectives for SBUs, allocation of re-sources among functional areas and coordination between them for making optimal contribution to the achievement of corporate-level objectives. Such strategies operate within the overall strategies of the organization. The corporate strategy sets the long-term objectives of the firm and the broad constraints and policies within which a SBU operates. The corporate level will help the SBU define its scope of operations and also limit or enhance the SBUs operations by the resources the corporate level assigns to it. There is a difference between corporate-level and business-level strategies. For example, Andrews says that in an organization of any size or diversity, corporate strategy usually applies to the whole enterprise, while business strategy, less comprehensive, defines the choice of product or service and market of individual business within the firm. In other words, business strategy relates to the how and corporate strategy to the what. Corporate strategy defines the business in which a company will compete preferably in a way that focuses resources to convert distinctive competence into competitive advantage. Corporate strategy is not the sum total of business strategies of the corporation but it deals with different subject matter. While the corporation is concerned with and has impact on business strategy, the former is concerned with the shape and balancing of growth and renewal rather than in market execution. The differences between corporate level strategy and business level strategy are noteworthy as each influences the performance of an organization. There are three commonly recognized levels of strategy within any corporation. The main identifier of each level is its sphere of influence within the organization. While the three levels are interrelated in many ways, each
44

must be separately considered to achieve the peak performance levels of which the business is capable. corporate and business strategies work together and influence each other in an effort to make the business units and the corporation successful. Small businesses engaged in a single industry already have made the only corporate-level strategic decision they have which industry to join. Small businesses contemplating diversification, on the other hand, face a raft of additional corporate-strategy decisions, as well as business-level decisions for the new business unit, should it decide to diversify. Our start-up candy makers corporate decision was to enter the confectionary market. Her business decisions were based on how to compete, which in turn influence her operational strategies concerning distribution, manufacturing, promotion, price etc.. When she diversifies, the addition of another unit necessitates businesslevel decisions for the new unit. But it may also require a rethinking of the original candy making operations business strategy.

45

UNIVERSITY PAPER SOLUTION YEAR- 2011

46

1)Define Management control system. Which level of Managers is involved in it? How does MCS differ from Simpler Control processes?

Ans. Management control systems are methods of collecting information that are used to guide and

direct the behavior of staff members and management in order to achieve a company's goals. A management control system may use a variety of techniques to evaluate various areas to improve performance and productivity. Some areas addressed by management control systems may include accounting methods, employee incentive programs and performance measurement. Simons (1994) defined MCS as "the formal, information-based routines and procedures managers use to maintain or alter patterns in organizational activities". Accounting methods are often implemented and evaluated as part of a management control system. To control financial activities within a company, the area may be broken down into financial and managerial accounting. Financial accounting generally focuses on internal issues, such as reporting sales costs, while managerial accounting may focus on methods for determining product costs. While both areas cover business accounting issues, their methods of application generally differ, and separate systems implemented by a management control system may aid in ensuring reports remain accurate and impartial. Managerial accounting is typically responsible for providing management with information on controlling costs and improving the production process. Managerial accountants may also provide cost information on new products, make pricing decisions and monitor actual and budgetary costs. General financial accounting within management control systems aims to focus on a company's internal accounting issues. Financial accounting typically handles payroll and human resource issues affecting employees within the company. Accounts in this area may also manage employee costs and reimbursements under a control system. A company's department directors also may benefit from the organization provided by management control systems. Directors often are responsible for coordinating activities with human resources to create employee incentives and to hire upper-level managers. Under a

management control system, directors can better analyze production progress, provide appropriate job assignments and more effectively communicate with all company employees. Measuring performance also is an important component of management control systems. Directors and upper-level managers generally aim to provide fair and effective employee

evaluations. Control systems help reviewers develop performance evaluations that provide useful and encouraging employee insight and accurate productivity analysis; these evaluations may even be used as an opportunity to provide performance-based incentives. 47

Complete and accurate performance evaluations may be one of the most important methods of determining an employee's strengths and retaining the most competitive and efficient employees. Management control systems allow for flexibility and outside factors to affect the evaluation process. For example, if external extenuating circumstances negatively affect sales or productivity, an evaluation may account for this factor and include this as part of the evaluation process. There are, however, significant differences between the management control process and the simpler processes describes earlier: 1. Unlike in the thermostat or body temperature systems, the standard is not preset. Rather, it is a result of a conscious planning process. In this process, management decides what the organization should be doing, and part of the control process is a comparison of actual accomplishments with these plans. Thus, the control process in an organization involves planning. In many situations, planning and control can be viewed as two separate activities. Management control, however, involves both planning and control. 2. Like controlling an automobile (but unlike regulating room or body temperature),management control is not automatic. Some detectors in an organization may ne mechanical, but the manager often detect important information with their own eyes, ears, and other senses. Although she may have routing ways of comparing certain reports of what is happening with standards of what should be happening, the manager must personally perform the assessor function, deciding for herself whether the difference between actual and standard performance is significant enough to warrant action, and if so, what action to take. Then because actions intended to alter an organization behavior involve human beings, the manager must interact with at least one other person to effect change. 3. Unlike controlling an automobile, a functions performed by a single individuals, management control requires coordination among individuals An organization consists of many separate parts a need that exists only minimally in the case part works in harmony with the others a need that exists only minimally in the case of the various organs that control body temperature and not at all in the case of the thermostat. 4. The connection from perceiving the need for action to determining the action requires obtaining the desired result may not be clear. 5. Much management control is self-control that is control is maintained not by an external regulating device like the thermostat, but by manager who are using their own judgment rather than following instructions from superior. Drivers who obey the 65 mph speed limit do so not because a sign commands to, but because they have consciously decide that it is in their interest to obey the law.

48

1) Briefly describe Responsibility center, Engineered Expense center, Discretionary Expense center, Revenue center, profit center. How is the performance of the Head of this center evaluated?

Ans. Responsibility Centers: A responsibility center is an organization unit that is headed by a manager who is responsible for its activities and results. In Responsibility Accounting, revenues and costs information are collected and reported by responsibility centers. A decentralized environment results in highly dispersed decision making. As a result, it is imperative to monitor and judge the effectiveness of each manager.

Engineered Expense Center. Engineered cost is those for which the right or proper amount can be estimated with reasonable reliability, for example material and direct labor. It have the following characteristic 1. Their input can be measured in monetary terms 2. Their output can be measured in physical terms 3. The optimum dollar amount of input required to produce one unit of output can be determined. For engineered expense center, it decides whether the proposed operating budget represents s the unit cost of performing its task efficiently. In discretionary expense center, budgeting is determined by the magnitude of the job that needs to be done, it usually carried out in one of two ways: Incremental Budgeting; in this model, discretionary expense centers current level of expense is taken as starting point. This amount is adjusted for inflation, anticipated in the workload continuing job, special job, or the cost of compatible jobs in similar units (if available). Zero Base Budgeting; In this approach, budgeting made thorough analysis of each discretionary expense center on a rolling schedule, all are reviewed in every period (at least five year). Discretionary Expense Center: Staff units, including general and administrative (G&A) departments, such as finance, human resources, and legal; research and development (R&D) departments; and marketing units such as those performing advertising and promotion, are usually treated as discretionary expense centers. The output from these units is not easily measured in 49

financial terms, and the relationship between the resources they expend (inputs) and the outcomes they produce is weak. Companies control these discretionary expense centers by negotiating and eventually authorizing an annual budget and then monitoring whether their actual spending remains within the budgeted amounts. Some organisation units have outputs that are not measured in monetary terms.These are principally administrative staff units (e.g accounting,legal,industrial

relations),research and product development organisation, and some types of marketing activities, Management makes budgetary decisions for discretionary expense center that differ from those foe engineered expense center. Management formulates the budget for discretionary expense center by determining the magnitude of the job that needs to be done. The work done by discretionary expense center falls in to two general categories: Continuing work is done consistently from year to year, such as the preparation of financial statement by the controller office. Special work is a one-shot project-for eg. Developing and installing a profit-budgeting system in a newly acquired division. A technique often used in preparing a discretionary expense centers budget is management by objectives, a formal process in which a budgeted proposes to accomplish specific jobs and suggests the measurement to be used in performance evaluation. The primary job of the discretionary expense centers manager is to obtain the desired output. Spending an amount that is on budget to do this is considered satisfactory; spending more than that is cause for concern; and spending less may indicate that the planned work is not being done. In discretionary center as opposed to engineered centers, the financial performance report is not a means of evaluating the efficiency of the manager. If these two types of responsibility centers are not carefully distinguished, management may erroneously treat a discretionary expense centers performance report as an indication of the units efficiency, thus motivating those making spending decision to expand less than the budgeted amount, which in turn will lower output. For this reason, it is unwise to reward executive who spend less than the budgeted amount. a. Revenue Center: A manager of a revenue center is held accountable for the revenue attributed to the sub-unit. Revenue centers are responsibility centers where managers are accountable only for financial outputs in the form of generating sales revenue. A revenue center's manger may also be held accountable for selling expenses such as sales persons' salaries, commissions, and order receiving costs.

50

b. The Profit Center: Many operating unit managers have responsibility and authority for both production and sales. They make decisions about what products and services to produce, how to produce them, their quality level, price, sales and distribution systems. But these managers may not have the authority to determine the level of capital investment in their facilities. In these cases, operating profit may be the single best (short- term) performance measure for how well the managers are creating value from the resources the company has put at their disposal. Such a unit, in which the manager has almost complete operational decision-making responsibility and is evaluated by a straightforward profit measure, is called a profit center. To aid performance evaluation it is first necessary to consider the specific character of each responsibility center. Some responsibility centers are cost centers and others are profit centers. On a broader scale, some are considered to be investment centers. The logical method of assessment will differ based on the core nature of the responsibility center. Basic devices are essential to a well managed organization. Managers must be held accountable for the results of their decisions and related execution. Without performance-related feedback, the business will not perform at its best possible level, and opportunities for improvement may go unnoticed. Given that managers must be held accountable for decisions, actions, and outcomes, it becomes very important to align a manager's area of accountability with their area of responsibility. The "area" of responsibility can be a department, product, plant, territory, division, or some other type of unit or segment. Usually, the attribution of responsibility will mirror the organizational structure of the firm. This is especially true in organizations that have a decentralized approach to decision-making. 2)Every SBU is a profit center but every profit center may not be SBU. Explain. Under what conditions production, Marketing and Service Department are converted into profit Center. Ans. In the competitive market environment of todays business cannot survive unless there is total accountability and associated responsibility and authority. Distribution sector also needs to be treated as a business entity if financial viability is to be achieved. The heads of the business units should be empowered to act and be held accountable for their actions & performance. Such a concept would be achievable if each circle is declared as a profit center with its own accounting system. The performance parameters as well as benchmarks can be set for improvement. This would also bring in the sense of ownership and competition, which are essential ingredients for success of a business. The MOA stresses upon the need for declaration of a circle as a profit center and establishing base line parameters as well as bench marks for measuring improvements consequent upon the commercial, administrative and technical interventions. The operating expenses of the circle, which contribute towards the delivery cost of energy to the customer, can also be monitored more closely as a profit center concept and measures may be initiated for reduction of the same.

51

Most business units are created as profit centers since managers in charge of such units typically control product development, manufacturing & marketing resources. these managers are in a position to influence revenues and costs and as such can be held accountable for the bottom line. However, a business unit managers authority may be constrained in various ways, which ought to be reflected in a profit centers design and operation. Conditions for an organization unit to be converted into a profit center: Many management decisions involve proposals to increase expenses with the expectation of an even greater increase in sales revenue. Such decisions are said to involve expense/revenue tradeoffs additional advertising expense is an example, before it is safe to delegate such a trade-off decision to a lower-level manager, two conditions should exist. 1. The manager should have access to the relevant information needed for making such a decision. 2. There should be some way to measure the effectiveness of the trade-offs the managers has made. A major step in creating profit centers is to determine the lowest point in an organization where these two conditions prevail. All responsibility centers fit into a continuum ranging from those that clearly should be profit centers to those that clearly should not, management must decide whether the advantages of giving profit responsibility offset the disadvantages, which are discussed below .As with all management control system design choices; there is no clear line of demarcation.

2) When are Market based transfer prices most appropriate? How do we deal with the condition of Limited Market, Situation of excess/shortage of capacity?

Ans. Transfer price if designed appropriately has the following objectives: It should provide each segment with the relevant information required to determine the optimum trade-off between company costs and revenues. It should induce goal congruent decisions-i.e. the system should be so designed that decisions that improve business unit profits will also improve company profits. It should help measure the economic performance of the individual profit centers. The system should be simple to understand and easy to administer.

52

The ideal transfer price in the situations of Limited Market

By limited market it means that the markets for buying and selling profit centers may be limited. Even in case of limited market the transfer price that is ideal or satisfies the requirement of a profit center system is the competitive price. In case if a company is not buying or selling its product in an outside market there are some ways to find the competitive price. They are as follows: If published market prices are available, they can be used to establish transfer prices. However, these should be prices actually paid in the market-place and the conditions that exist in the outside market should be consistent with those existing within the company. For example, market prices that are applicable to relatively small purchases are not valid in this case. Market prices are set by bids. This generally can be done only if the low bidder has a reasonable chance of obtaining the business. One company accomplishes this by buying about one-half of a particular group of products outside the company and one-half inside the company. The company then puts all of the products out to bid, but selects one-half to stay inside. The company obtains valid bids, because low bidders can expect to get some of the business. By contrast, if a company requests bids solely to obtain a competitive price and does not award the contracts to the low bidder, it will soon find that either no one bids or that the bids are of questionable value. If the production profit center sells similar products in outside markets, it is often possible to replicate a competitive price on the basis of the outside price. If the buying profit center purchases similar products from the outside market, it may be possible to replicate competitive prices for its proprietary products. This can be done by calculating the cost of the difference in design and other conditions of sale between the competitive products and the proprietary products.

Shortage of Capacity in the industry In this case, the output of the buying profit center is constrained and again company profits may not be optimum. Some companies allow either buying profit center to appeal a sourcing decision to a central person or committee. In this scenario a buying profit center could appeal a selling profit centers decision to sell outside. The person/group would then make a sourcing decision on the basis of the companys best interests. In every case the transfer price would be the competitive price. In other words, the profit center is appealing only the sourcing decision. 53

Even if there are constraints on sourcing, the market price is the best transfer price. If the market price can be approximated, it is ideal transfer price.

3) What do you understand by investment center? Explain two' different methods by which the performance of these centers are measured? Also discuss their relative merits and demerits?

Ans. An investment centre goes a step further than a profit centre does. Its success is measured not only by its income but also by relating that income to its invested capital, as in a ratio of income to the value of the capital employed. In practice, the term investment centre is not widely used. Instead, the term profit centre is used indiscriminately to describe centers that are always assigned responsibility for revenues and expenses, but may or may not be assigned responsibility for the capital investment. It is defined as a responsibility centre in which inputs are measured in terms of cost / expenses and outputs are measured in terms of revenues and in which assets employed are also measured. A responsibility centre is called an investment centre, when its manager is responsible for costs and revenues as well as for the investment in assets used by his centre. He is responsible for maintaining a satisfactory return on investment i.e. asset employed in his responsibility centre. The investment centre manager has control over revenues, expenses and the amounts invested in the centres assets. The manager of an investment centre is required to earn a satisfactory return. Thus, return on investment (ROI) is used as the performance evaluation criterion in an investment centre. He also formulates the credit policy, which has a direct influence on debt collection, and the inventory policy, which determines the investment in inventory. The Vice President (Investments) of a mutual funds company may be in charge of an Investment Centre. In the Investment Centre, the manager in charge is held responsible for the proper utilization of assets. He is expected to earn a satisfactory return on the assets employed in his responsibility centre. The following techniques are useful in evaluating the performance of an investment centre:

1. Return on investment (ROI): The rate of return on investment is determined by dividing net profit or income by the capital employed or investment made to achieve that profit. ROI = Profit / Invested capital * 100 ROI consists of two components viz. 54

Profit margin Investment turnover ROI = Net profit / Investment = (Net profit / Sales) * (Sales / Investment in assets) It will be seen from the above formula that ROI can be improved by increasing one or both of its components viz.the profit margin and the investment turnover in any of the following ways: Increasing the profit margin Increasing the investment turnover Increasing both profit margin and investment turnover Capital employed is taken to be the total of shareholders funds, loans etc The profit figure used is in calculating ROI is usually taken from the profit and loss account, profit arising out of the normal activities of the company should only be taken. Capital employed for the company as a whole can be arrived at as follows:

Share capital of the company Reserves and surplus Loans (secured/unsecured) xxx

xxx xxx

-----xxx Less: a. Investment outside the business b. Preliminary expenses c. Debit balance of P & L A/c xxx xxx xxx xxx ____ xxxx

55

Merits: Return on investment analysis provides a strong incentive for optimum utilization of the assets of the company. This encourages managers to obtain assets that will provide a satisfactory return on investment and to dispose off assets that are not providing an acceptable return. In selecting amongst alternative long-term investment proposals, ROI provides a suitable measure for assessment of profitability of each proposal.

Demerits: ROI analysis is not very suitable for short-term projects and performances. In the initial stages a new investment may yield a small ROI which may mislead the management. Most likely the rate would improve in course of time when the initial difficulties are overcome.

The book value of assets decline due to depreciation, the investment base will continuously decrease in value, causing the rate of return to increase.

2. Residual income: Residual income can be defined as the operating profit (or income) of the company less the imputed interest on the assets used by the company. In other words, interest on the capital invested in the company is treated as a cost and any surplus is the residual income. Residual income is profit minus notional interest charge on capital employed. Residual income is affected by the size of the organization and therefore will not provide a basis for evaluation of organizational performance. This is probably the main reason why the management continues to make use of ROI which is relative measure. Not all projects start off with positive or sufficiently large positive profits in the early years of a project to produce a positive increment to residual income. It has been argued that a more suitable measure of performance for investment centers, which could encourage managers to be more willing to undertake marginally profitable projects, is residual income.

56

Advantages: 1) It considers a company's minimum rate of return. 2) Any project that increases residual income will be pursued by division management.

Disadvantage: The relative size of the divisions is not considered.

We recommend RI as a method of evaluating performance of an investment centre. Because when RI is adopted for evaluation purposes, emphasis is placed on marginal profit amount above the cost of capital rather than on the rate itself.

4) Briefly describe Functional, Divisional and Matrix organization. Which is the most appropriate from the point of Control? Where are the other two suitable?

Ans. Functional Organization:The organization is divided into segments based on the functions when managing. This allows the organization to enhance the efficiencies of these functional groups. As an example, take a software company. Software engineers will only staff the entire software development department. This way, management of this functional group becomes easy and effective. Functional structures appear to be successful in large organization that produces high volumes of products at low costs. The low cost can be achieved by such companies due to the efficiencies within functional groups. In addition to such advantages, there can be disadvantage from an organizational perspective if the communication between the functional groups is not effective. In this case, organization may find it difficult to achieve some organizational objectives at the end.

57

Divisional Organization:These types of organizations divide the functional areas of the organization to divisions. Each division is quipped with its own resources in order to function independently. There can be many bases to define divisions. Divisions can be defined based on the geographical basis, products / services basis, or any other measurement. As an example, take a company such as General Electrics. It can have microwave division, turbine division, etc., and these divisions have their own marketing teams, finance teams etc. In that sense, each division can be considered as a micro-company with the main organization.

Matrix Organization:When it comes to matrix structure, the organization places the employees based on the function and the product. The matrix structure gives the best of the both worlds of functional and divisional structures. In this type of an organization, the company uses teams to complete tasks. The teams are formed based on the functions they belong to (ex: software engineers) and product they involved in (ex: Project A). This way, there are many teams in this organization such as software engineers of project A, software engineers of project B, QA engineers of project A, etc.

Every organization needs a structure in order to operate systematically. The organizational structures can be used by any organization if the structure fits into the nature and the maturity of the organization. In most cases, organizations evolve through structures when they progress through and enhance their processes and manpower. One company may start as a pre-bureaucratic company and may evolve up to a matrix organization. 58

5) What do you understand by a Not for Profit organization? How does this organization price their products? What criteria are used to measure their performance? Ans. A nonprofit organization, as defined by law, is an organization that cannot distribute assets or income to, or for the benefit of, its members, officers, or directors. The organization can, of course, compensate its employees, including officers and members, for services rendered and for goods supplied. This definition does not prohibit an organization from earning a profit; it prohibits only the distribution of profits. A nonprofit organization needs to earn a modest profit on average, to provide funds for working capital and for possible rainy days.

Performance evaluation of nonprofit organization For any organization, the most important reasons to measure performance are to improve effectiveness and to acquire information that will allow the organization to drive its agenda forward. If the motivation for doing evaluation remains outside an organization, the evaluation will have limited impact. To do performance assessment effectively, an organization must commit to adopting a culture of measurement, because acceptance must come from senior management, staff, funders, and board members alike.

Board self-evaluation: Members of the Board of Directors should regularly evaluate the quality of their activities on a regular basis. Activities might include staffing the Board with new members, developing the members into well-trained and resourced members, discussing and debating topics to make wise decisions, and supervising the CEO. Probably the biggest problem with Board selfevaluation is that it does not occur frequently enough. As a result, Board members have no clear impression of how they are performing as members of a governing Board. Poor Board operations, when undetected, can adversely affect the entire organization.

Staff and volunteer (individual) performance evaluation:Most of us are familiar with employee performance appraisals, which evaluate the quality of an individuals performance in their position in the organization. Ideally, those appraisals reference the individuals written job description and performance goals to assess the quality of the individuals progress toward achieving the desired results described in those documents. Continued problems in individual performance often are the results of poor strategic planning, program planning and staff development. If overall planning is not done effectively, individuals can experience continued frustration, stress and low morale, resulting in their poor overall performance. Experienced leaders have learned that continued problems in performance are not always the result of a poor work ethic the recurring problems may be the result of larger, more systemic problems in the organizations.

59

Program evaluation: Program evaluations have become much more common, particularly because donors demand them to ensure that their investments are making a difference in their communities. Program evaluations are typically focused on the quality of the programs process, goals or outcomes. An ineffective program evaluation process often is the result of poor program planning programs should be designed so they can be evaluated. It can also be the result of improper training about evaluation. Sometimes, leaders do not realize that they have the responsibility to verify to the public that the nonprofit is indeed making a positive impact in the community. When program evaluations are not performed well, or at all, there is little feedback to the strategic and program planning activities. When strategic and program planning are done poorly, the entire organization is adversely effected.

Evaluation of cross-functional processes: Cross-functional processes are those that span several systems, such as programs, functions and projects. Common examples of major processes include information technology systems and quality management of services. Because these cross-functional processes span so many areas of the organization, problems in these processes can be the result of any type of ineffective planning, development and operating activities.

Organizational evaluation: Ongoing evaluation of the entire organization is a major responsibility of all leaders in the organization. Leaders sometimes do not recognize the ongoing activities of management to actually include organizational evaluations but they do. The activities of organizational evaluation occur every day. However, those evaluations usually are not done systematically. As a result, useful evaluation information is not provided to the strategic and program planning processes. Consequently, both processes can be ineffective because they do not focus on improving the quality of operations in the workplace. Q What are organization structure and Management control implication of corporate Level strategies? 6) Write short notes on any two of the following :

(a) Approach to Marketing by professional service organization

Ans. In a manufacturing company there is a clear dividing line between marketing activities and production activities; only senior management is concerned with both. Such a clean separation does not exist in most professional organizations. In some, such as law, medicine, and accounting, the profession's ethical code limits the amount and character of overt marketing efforts by professionals (although these restrictions have been relaxed in recent years). Marketing is an essential activity in 60

almost all organizations, however. If it can't be conducted openly, it takes the form of personal contacts, speeches, articles, conversations on the golf course, and so on. These marketing activities are conducted by professionals, usually by professionals who spend much of their time in production work-that is, working for clients. In this situation, it is difficult to assign appropriate credit to the person responsible for "selling" a new customer. In a consulting firm, for example, a new engagement may result from a conversation between a member of the firm and an acquaintance in a company, or from the reputation of one of the firm's professionals as an outgrowth of speeches or articles. Moreover, the professional who is responsible for obtaining the engagement may not be personally involved in carrying it out. Until fairly recently, these marketing contributions were rewarded subjectively- that is, they were taken into account in promotion and compensation decisions. Some organizations now give explicit credit, perhaps as a percentage of the project's revenue, if the person who "sold" the project can be identified.

(b) Balance Score Card Ans. The rationale for the development of the Balanced Scorecard was a growing dissatisfaction with traditional, financial measures of performance. These measures suffer from a number of serious drawbacks in that they take a short-term, lagged (i.e., historic) view of performance. The shift towards flexible, lean production/service systems in many firms has strengthened the requirement for performance measurement systems to become more broadly based, incorporating both non-financial and external measures of performance. According to Kaplan and Norton, the Balanced Scorecard provides a better assessment of performance as it "enables companies to track financial results while simultaneously monitoring progress in building the capabilities and acquiring the intangible assets they need for future growth". The original scorecard designed by Kaplan and Norton contained four key groupings of performance measures. These four groupings, called perspectivesby Kaplan and Norton, were considered sufficient to track the key drivers of both current and future financial performance of the firm. The perspectives focused on the achievements of the firm in four areas: namely the financial, customer, internal business process and innovation/learning perspectives. The four perspectives can be represented as an interlinked hierarchy. The firms strategy underlies the whole scorecard, as the measures for each of the four perspectives are drawn from this strategy.

61

To obtain a satisfactory overview of performance, the scorecard will require a mix of lagging and leading (forward looking) measures. Financial measures tend to be lagged and consequently, the measures chosen for the other perspectives will need to include leading measures. In general, outcome measures tend to be lagged, for example, current market share is the result of past decisions and consequently is a lagging measure. Thus the challenge in designing a Balanced Scorecard is to choose driver measures which lead changes in the outcome measures in the non-financial perspectives and which ultimately drive the financial measures. Once the firms objectives have been agreed and the appropriate outcome and driver measures chosen for each of the perspectives, firm and managerial performance is assessed by comparing actual attainment on each measure with the target set for that measure.

(c) Interactive Controls.

Ans.Interactive control is a subset of the management control information that has a bearing on the strategic uncertainties facing the business becomes the focal point. In industries that are subject to very rapid environmental changes, management control information can also provide the basis for thinking about new strategies. In a rapidly changing and dynamic environment, creating a learning organization is essential to corporate survival. Learning organization refers to the ability of organization employees to learn to cope with environmental changes on an ongoing basis. An effective learning organization is one in which employees at all levels continuously scan the environment, identify potential problems and opportunities, exchange environment information candidly and openly and experiment with alternative environment. The main objective of interactive control is to facilitate the creation of a learning organization. Interactive controls alert management to strategic uncertainties, either troubles (e.g., loss of market share, customer complaints) or opportunities (e.g., opening a new market because certain

62

governmental regulations have been removed). These become the basis for managers to adapt to a rapidly changing environment by thinking about new strategies.

Interactive control has the following characteristics: 1) A subset of the management control information that has a bearing on the strategic uncertainties facing the business becomes the focal point. 2) Senior executives take such information seriously. 3) Managers at all levels of the organization focus attention on the information produced by the system. 4) Superiors, subordinates and peers meet face-to-face to interpret and discuss the implications of the information for future strategic 5) The face-to-face meetings take the form of debate and challenge of the underlying data, assumptions, and appropriate actions. Strategic uncertainties relate to fundamental, nonlinear shifts in the environment that potentially can create new business models. Firms should monitor the following technological discontinuities: 1. Internet and e-commerce growth have potential implication for many firms. Some of the particular items to monitor include: Growth in the number of internet users. Roll-out of broadband communications. Emergence of ubiquitous point-and-click interfaces that are based on open standards, cheap to set up and run, and global. Increasing power of computing and communication technologies. Growth in mobile communications for both voice telephony and internet access. Development and deployment of speech recognition and machine based language translation technologies that may make it possible for people speaking or writing different languages to communicate with each other in real time. 2. Converging technologies will have the following effects: Convergence of voice, data, and image has implications for firms operating in consumer electronics (Phillips), telecommunications (British Telecom), and computer (IBM) industries. Integration of chemical and digital technologies has impact on firms such as Eastman Kodak. Blending of hardware and software has impact on firms such as Sony. Merging of plant engineering and biotechnology opens up opportunities for firms I life sciences (Novartis, Merck, Pfizer). 63

3. Shift from physical goods to services is rapidly transforming the automobile industry (Ford) and consumer durable goods business (General Electric). Interactive controls are not a separate system; they are an integral part of the management control system. Some management control information helps managers think about new strategies. Interactive control information usually, but not exclusively, tends to be nonfinancial. Since strategic uncertainties differ from business to business, senior executives in different companies might choose different parts of their management control system to use interactively.

64

Anda mungkin juga menyukai